Learn more

Refine search


Results (12 matches)

  displayed columns for results
Label Class Conductor Rank Torsion CM Regulator Weierstrass coefficients Weierstrass equation mod-$m$ images
5010.f1 5010.f \( 2 \cdot 3 \cdot 5 \cdot 167 \) $1$ $\mathsf{trivial}$ $0.350671163$ $[1, 1, 1, 24, 153]$ \(y^2+xy+y=x^3+x^2+24x+153\) 1670.2.0.?
15030.h1 15030.h \( 2 \cdot 3^{2} \cdot 5 \cdot 167 \) $1$ $\mathsf{trivial}$ $1.670591521$ $[1, -1, 0, 216, -3920]$ \(y^2+xy=x^3-x^2+216x-3920\) 1670.2.0.?
25050.f1 25050.f \( 2 \cdot 3 \cdot 5^{2} \cdot 167 \) $2$ $\mathsf{trivial}$ $0.183351174$ $[1, 0, 1, 599, 17948]$ \(y^2+xy+y=x^3+599x+17948\) 1670.2.0.?
40080.x1 40080.x \( 2^{4} \cdot 3 \cdot 5 \cdot 167 \) $1$ $\mathsf{trivial}$ $0.506675507$ $[0, 1, 0, 384, -9036]$ \(y^2=x^3+x^2+384x-9036\) 1670.2.0.?
75150.bd1 75150.bd \( 2 \cdot 3^{2} \cdot 5^{2} \cdot 167 \) $0$ $\mathsf{trivial}$ $1$ $[1, -1, 1, 5395, -484603]$ \(y^2+xy+y=x^3-x^2+5395x-484603\) 1670.2.0.?
120240.bk1 120240.bk \( 2^{4} \cdot 3^{2} \cdot 5 \cdot 167 \) $1$ $\mathsf{trivial}$ $1.694714050$ $[0, 0, 0, 3453, 247426]$ \(y^2=x^3+3453x+247426\) 1670.2.0.?
160320.t1 160320.t \( 2^{6} \cdot 3 \cdot 5 \cdot 167 \) $1$ $\mathsf{trivial}$ $2.542815989$ $[0, -1, 0, 1535, -73823]$ \(y^2=x^3-x^2+1535x-73823\) 1670.2.0.?
160320.cz1 160320.cz \( 2^{6} \cdot 3 \cdot 5 \cdot 167 \) $0$ $\mathsf{trivial}$ $1$ $[0, 1, 0, 1535, 73823]$ \(y^2=x^3+x^2+1535x+73823\) 1670.2.0.?
200400.z1 200400.z \( 2^{4} \cdot 3 \cdot 5^{2} \cdot 167 \) $1$ $\mathsf{trivial}$ $0.963131987$ $[0, -1, 0, 9592, -1148688]$ \(y^2=x^3-x^2+9592x-1148688\) 1670.2.0.?
245490.by1 245490.by \( 2 \cdot 3 \cdot 5 \cdot 7^{2} \cdot 167 \) $1$ $\mathsf{trivial}$ $0.745197851$ $[1, 0, 0, 1175, -49015]$ \(y^2+xy=x^3+1175x-49015\) 1670.2.0.?
480960.bc1 480960.bc \( 2^{6} \cdot 3^{2} \cdot 5 \cdot 167 \) $0$ $\mathsf{trivial}$ $1$ $[0, 0, 0, 13812, 1979408]$ \(y^2=x^3+13812x+1979408\) 1670.2.0.?
480960.ch1 480960.ch \( 2^{6} \cdot 3^{2} \cdot 5 \cdot 167 \) $0$ $\mathsf{trivial}$ $1$ $[0, 0, 0, 13812, -1979408]$ \(y^2=x^3+13812x-1979408\) 1670.2.0.?
  displayed columns for results